LSAT 106 – Section 1 – Question 26
You need a full course to see this video. Enroll now and get started in less than a minute.
Target time: 1:17
This is question data from the 7Sage LSAT Scorer. You can score your LSATs, track your results, and analyze your performance with pretty charts and vital statistics - all with a Free Account ← sign up in less than 10 seconds
Question QuickView |
Type | Tags | Answer Choices |
Curve | Question Difficulty |
Psg/Game/S Difficulty |
Explanation |
---|---|---|---|---|---|---|---|
PT106 S1 Q26 |
+LR
| Except +Exc Strengthen +Streng Causal Reasoning +CausR | A
1%
161
B
5%
163
C
5%
160
D
6%
161
E
83%
168
|
147 155 163 |
+Harder | 152.148 +SubsectionHarder |
Summarize Argument: Phenomenon-Hypothesis
The author hypothesizes that passengers using electronic devices put modern airplanes at risk. Why? Because the planes’ navigation systems use low-power circuitry that’s more susceptible to interference than vacuum-tube circuitry in old planes, as illustrated by a recent incident where a navigation system apparently malfunctioned when a passenger opened a laptop.
Notable Assumptions
The author assumes the navigation system’s behavior during the off-course landing was unusual and occurred because the passenger opened their laptop, and not for some other reason. This means assuming laptops and cassette players carried by passengers produce radiation that’s capable of interfering with a modern navigation system.
A
After the laptop computer was turned off, the plane regained course and its navigation instruments and dials returned to normal.
This strengthens the argument by making it more likely the laptop caused the navigation system’s behavior during the off-course landing.
B
When in use all electronic devices emit electromagnetic radiation, which is known to interfere with circuitry.
This strengthens the argument by confirming that cassette players, laptops, and other devices produce radiation. It rules out the possibility that some electronic devices emit no radiation, which would weaken the argument.
C
No problems with navigational equipment or instrument dials have been reported on flights with no passenger-owned electronic devices on board.
This strengthens the argument by ruling out a powerful counterexample. If similar behavior had occurred on flights with no passenger-owned devices, then the laptop’s responsibility would be less likely.
D
Significant electromagnetic radiation from portable electronic devices can travel up to eight meters, and some passenger seats on modern aircraft are located within four meters of the navigation systems.
This strengthens the argument by confirming that passengers sit close enough for radiation from their devices to reach navigation systems. If all the passengers sat too far away for their radiation to reach those systems, the argument would falter.
E
Planes were first equipped with low-power circuitry at about the same time portable electronic devices became popular.
This is irrelevant. The author concludes there’s a risk presently—it doesn’t matter when passenger-owned devices and low-power circuitry became prevalent, so long as they’re both prevalent right now.
Take PrepTest
Review Results
LSAT PrepTest 106 Explanations
Section 1 - Logical Reasoning
- Question 01
- Question 02
- Question 03
- Question 04
- Question 05
- Question 06
- Question 07
- Question 08
- Question 09
- Question 10
- Question 11
- Question 12
- Question 13
- Question 14
- Question 15
- Question 16
- Question 17
- Question 18
- Question 19
- Question 20
- Question 21
- Question 22
- Question 23
- Question 24
- Question 25
- Question 26
Section 2 - Logical Reasoning
- Question 01
- Question 02
- Question 03
- Question 04
- Question 05
- Question 06
- Question 07
- Question 08
- Question 09
- Question 10
- Question 11
- Question 12
- Question 13
- Question 14
- Question 15
- Question 16
- Question 17
- Question 18
- Question 19
- Question 20
- Question 21
- Question 22
- Question 23
- Question 24
- Question 25
- Question 26
Section 3 - Logical Reasoning
- Question 01
- Question 02
- Question 03
- Question 04
- Question 05
- Question 06
- Question 07
- Question 08
- Question 09
- Question 10
- Question 11
- Question 12
- Question 13
- Question 14
- Question 15
- Question 16
- Question 17
- Question 18
- Question 19
- Question 20
- Question 21
- Question 22
- Question 23
- Question 24
- Question 25
- Question 26
Leave a Reply
You must be logged in to post a comment. You can get a free account here.